Is it possible that a particle is much heavier through a loop correction

In summary, the conversation discusses a scenario where standard model singlet particles mix with an exotic, vectorlike neutral lepton N after electroweak symmetry breaking. The Lagrangian for this scenario does not include a Majorana mass term for the singlet s at the tree level, but it can be generated at the 1-loop level through a diagram with N in the loop. The resulting correction to the singlet s's mass is comparable to the tree-level estimate, divided by a loop factor, and possibly multiplied by a logarithm. This is similar to the usual seesaw scenario for left-handed neutrinos, where the singlet νR is heavy and the left-handed neutrinos νL are light. However, in this scenario
  • #1
jakob1111
Gold Member
26
17
Let's assume, we have standard model singlet particle s that mixes after electroweak symmetry breaking with an exotic, vectorlike neutral lepton N The relevant part of the Lagrangian reads

$$ L \supset h^c s N + h s N^c + M N N^c, $$

where h is the standard model higgs and M is a superheavy mass. Moreover, we assume that for some reason there is (at tree level) no Majorana mass term: $$M_s ss$$ for the singlet s. The tree-level analysis now yields for the singlet s$a tiny seesaw type mass: $$m_s \approx v_{EW} / M^2 \ll v_{EW}$$.

Now, a Majorana mass term for the singlet s will be generically generated at the 1-loop level through a diagram with N in the loop. It was pointed out to me that this 1-loop contribution "may give rise to a much larger mass for the singlet". I would like understand how this can happen. I think the relevant diagram looks like this

PEKrA.png


My naive estimate for this one-loop contribution is $$ m_s \approx 1/16 \pi \ m_{EW}^2 /M$$, i.e. something comparable to the tree-level estimate, divided by a loop factor, possibly times some logarithm. Thus, while there is possibly some relevant correction due to the logarithm, the consequences do not seem dramatic.

Is there any other possible correction that I'm missing here? Is there some diagram that potentially leads to a much heavier mass for the singlet s?----------A relevant analogous scenario

The situation is similar to the usual seesaw for the left-handed neutrinos νLνL. However, the situation described above is reversed. In the usual seesaw scenario, the left-handed neutrinos νLνL are light and the singlet νRνR is heavy. The 1-loop correction to the usual seesaw formula, is discussed in On the importance of the 1-loop finite corrections to seesaw neutrino masses by D. Aristizabal Sierra, Carlos E. Yaguna. (See also, this summary). The relevant diagrams are

yhTRQ.png


and the result is

apFMr.png


This result yields a contribution comparable to the result of the tree-level analysis: $$ m_{EW}^2 / M$$, where $$m_{EW}$$ denotes the electroweak scale and $$M$$ a superheavy scale. (In addition there is a potentially enhancing log factor.)
 
  • Like
Likes Kninfinite
Physics news on Phys.org
  • #2
Thus, the 1-loop contribution does not drastically change the tree-level result.The situation discussed above is analogous to the usual seesaw for νLνL. However, the situation is reversed: The singlet s is light and the vectorlike lepton NN is heavy. Thus, my naive expectation is that the 1-loop diagrams will yield a correction that is comparable to the tree-level result. Is this correct? Are there any diagrams that potentially lead to a much larger 1-loop correction?
 

1. Is it possible for a particle to become significantly heavier through a loop correction?

Yes, it is possible for a particle to become significantly heavier through a loop correction. This occurs when the particle interacts with virtual particles in a loop, which can add to its mass through the process of renormalization.

2. How does a loop correction affect the mass of a particle?

A loop correction can increase the mass of a particle by adding the contributions from virtual particles in the loop. This is a result of the renormalization process, which accounts for the effects of virtual particles on the observed mass of a particle.

3. Can a loop correction cause a particle to become infinitely heavy?

No, a loop correction cannot cause a particle to become infinitely heavy. The renormalization process ensures that the observed mass of a particle remains finite, even when considering the contributions from virtual particles in the loop.

4. What is the significance of loop corrections in particle physics?

Loop corrections are important in particle physics because they help to explain the observed masses of particles and provide a way to account for the effects of virtual particles. They also play a crucial role in quantum field theory calculations and are an integral part of the standard model of particle physics.

5. Are loop corrections the only factor that can affect the mass of a particle?

No, loop corrections are not the only factor that can affect the mass of a particle. Other factors, such as interactions with other particles and the Higgs field, can also contribute to the observed mass of a particle. However, loop corrections are an important factor to consider and can significantly impact the mass of a particle.

Similar threads

  • High Energy, Nuclear, Particle Physics
Replies
1
Views
1K
  • Beyond the Standard Models
Replies
9
Views
482
  • Advanced Physics Homework Help
Replies
2
Views
2K
  • Beyond the Standard Models
Replies
18
Views
3K
Replies
1
Views
2K
  • Beyond the Standard Models
Replies
28
Views
4K
Back
Top